Gradient & Directional Derivative Question (multi var)

In summary: So far so good?B) Find the rate of change of f at the point (√5, -2) in the direction toward the point (0,0).I'm going to use this equation:\frac
  • #1
Alex Bard
21
0
[itex]\partial[/itex]This is my first post, so I apologize for all my mistakes. Thank you for the help, in advance.
These are test review questions for Multi Variable Calculus.

Homework Statement



Let f(x,y) = tan-1(y2 / x)
a) Find fx([itex]\sqrt{5}[/itex], -2) and fy([itex]\sqrt{5}[/itex], -2).

b) Find the rate of change of f at the point ([itex]\sqrt{5}[/itex], -2) in the direction toward the point (0,0).

c) Find the directional derivative in the direction of fastest increase at the point ([itex]\sqrt{5}[/itex], -2).

Homework Equations



So from my understanding, part a, I would just have to do partial derivatives to x then y and then plug in the coordinates afterwards that are given.

Part b is the gradient vector.

Part c is just the largest value when getting the derivative.


The Attempt at a Solution



a. [itex]\frac{∂f}{∂x}[/itex] = [1 / 1 + (y^2/x)^2] * y^2 = (y^2 * x^2)/1+y^4
when subbing in the points given, I get 20/17

[itex]\frac{∂f}{∂y}[/itex] = 1 / [1 + (y^2/x)^2] * 2y/x = (2y * x^2) / x(1 + y^4)
when subbing in the points given, I get 20/17√5

b. So here I would first make a vector from (√5, -2) going to (0,0), which is <-√5, 2>
Then unit vector is 1/√[(√5)2 + 22] = 1 / √9
= 1/3

Then 1/3(-√5, 2) is the unit vector

To get the Gradient, I use ∂ in respect to x and y AND using the calculations from part a, I get:

∇f(x,y) = <(y^2 * x^2)/1+y^4, (2y * x^2) / x(1 + y^4)>

now I'm a bit stuck. Do I just do the DOT product of ∇f and u? as in:

1/3(-√5, 2) ° <(y^2 * x^2)/1+y^4, (2y * x^2) / x(1 + y^4)>

or should I sub the points given of (√5, -2) into the ∇f and then dot them?

I haven't really gotten to part c yes since I believe I need the answer from b to do c.

All your help is appreciated.
 
Physics news on Phys.org
  • #2
It's hard to read your work with lots of missing parentheses and, I think, some mistakes. But to answer your last question, I would say to evaluate both partials at the point and your gradient at the point. Then dot that simple vector with your unit vector in whatever direction you need. Again, I think you need to check your arithmetic in spots.
 
  • #3
Alex Bard said:
[itex]\partial[/itex]This is my first post, so I apologize for all my mistakes. Thank you for the help, in advance.
These are test review questions for Multi Variable Calculus.

Homework Statement



Let f(x,y) = tan-1(y2 / x)
a) Find fx([itex]\sqrt{5}[/itex], -2) and fy([itex]\sqrt{5}[/itex], -2).

b) Find the rate of change of f at the point ([itex]\sqrt{5}[/itex], -2) in the direction toward the point (0,0).

c) Find the directional derivative in the direction of fastest increase at the point ([itex]\sqrt{5}[/itex], -2).

Homework Equations



So from my understanding, part a, I would just have to do partial derivatives to x then y and then plug in the coordinates afterwards that are given.

Part b is the gradient vector.

Part c is just the largest value when getting the derivative.


The Attempt at a Solution



area. [itex]\frac{∂f}{∂x}[/itex] = [1 / 1 + (y^2/x)^2] * y^2 = (y^2 * x^2)/1+y^4
when subbing in the points given, I get 20/17

[itex]\frac{∂f}{∂y}[/itex] = 1 / [1 + (y^2/x)^2] * 2y/x = (2y * x^2) / x(1 + y^4)
when subbing in the points given, I get 20/17√5

b. So here I would first make a vector from (√5, -2) going to (0,0), which is <-√5, 2>
Then unit vector is 1/√[(√5)2 + 22] = 1 / √9
= 1/3

Then 1/3(-√5, 2) is the unit vector

To get the Gradient, I use ∂ in respect to x and y AND using the calculations from part a, I get:

∇f(x,y) = <(y^2 * x^2)/1+y^4, (2y * x^2) / x(1 + y^4)>

now I'm a bit stuck. Do I just do the DOT product of ∇f and u? as in:

1/3(-√5, 2) ° <(y^2 * x^2)/1+y^4, (2y * x^ 2) / x(1 + y^4)>

or should I sub the points given of (√5, -2) into the ∇f and then dot them?

I haven't really gotten to part c yes since I believe I need the answer from b to do c.

All your help is appreciated.

Your ##f_x## and ##f_y## are incorrect; go back and *very carefully* use the chain rule. In particular, you should not have (1+y^4) in the denominators; something else needs to replace that.

Also: use brackets; your expression (y^2 * x^2)/1+y^4 reads as
[tex] \frac{y^2 x^2}{1} + y^4 = x^2 y^2 + y^4[/tex]
when read using standard rules for parsing mathematical expressions. If you mean
[tex] \frac{y^2 x^2}{1+y^4}[/tex]
then use parentheses, like this: y^2 * x^2/(1 + y^4).
 
  • #4
Reworking Part A

Okay, thank you for your help, let me do this all over again. Also, I apologize for my sloppy writing, will do my best to be neat and organized.

Lets have a crack at this.

a. ∂f∂x = [itex]\frac{1}{1+[\frac{y^2}{x}]^2} * y^2 * \frac{-1}{x^2}[/itex]

so : [itex]\frac{y^2 x^2} {(-x)^2 - y^4 x^2}[/itex]

Simplifying, I get : [itex]\frac{1} {-x^2 - y^2}[/itex]

Subbing in my values: [itex]\frac{1}{(-√5)^2 - 2^2}[/itex] = 1


∂f∂y = [itex]\frac{1}{1+[\frac{y^2}{x}]^2} * \frac{2y}{x}[/itex]

so : [itex]\frac{2yx^2} {x (1 + y^4)}[/itex]

Simplifying, I get : [itex]\frac{2yx^2} {x + xy^4}[/itex]

Now with substitution, I would get [itex]\frac{20}{17√5}[/itex]


Please let me know if I'm headed in the right direction. Thank you for your help.
 
  • #5
Alex Bard said:
Okay, thank you for your help, let me do this all over again. Also, I apologize for my sloppy writing, will do my best to be neat and organized.

Lets have a crack at this.

a. ∂f∂x = [itex]\frac{1}{1+[\frac{y^2}{x}]^2} * y^2 * \frac{-1}{x^2}[/itex]

so : [itex]\frac{y^2 x^2} {(-x)^2 - y^4 x^2}[/itex]

Simplifying, I get : [itex]\frac{1} {-x^2 - y^2}[/itex]

Check your simplification. Only the first line is correct.

Subbing in my values: [itex]\frac{1}{(-√5)^2 - 2^2}[/itex] = 1∂f∂y = [itex]\frac{1}{1+[\frac{y^2}{x}]^2} * \frac{2y}{x}[/itex]

so : [itex]\frac{2yx^2} {x (1 + y^4)}[/itex]

Simplifying, I get : [itex]\frac{2yx^2} {x + xy^4}[/itex]

Not wrong, but why not finish simplifying? Woops, yes it is wrong. Your algebra needs some work.
 
  • #6
Finished?

Thank you for the help LCKurtz, I'm going to go through the entire problem again, with feeling this time.


[itex] Let f(x,y) = tan^-1(\frac{y^2}{x})[/itex]

A) Find fx(√5, -2) and fy(√5, -2).

[itex]\frac{∂f}{∂x} = \frac{1}{1 + [\frac{y^2}{x}]^2} * y^2 * \frac{-1}{x^2}[/itex]

Since I suck at simplification, I will plug my numbers in right away to avoid confusion:

[itex]\frac{∂f}{∂x} = \frac{1}{1 + [\frac{(-2)^2}{√5}]^2} * (-2)^2 * \frac{-1}{(√5)^2}[/itex]

Solving:

[itex]\frac{∂f}{∂x} = \frac{1}{\frac{5}{5} + [\frac{16}{5}]} * 4 * \frac{-1}{5}[/itex]

→ [itex]\frac{∂f}{∂x} = \frac{-4}{21} [/itex]

Now for [itex]\frac{∂f}{∂y} [/itex]

[itex]\frac{∂f}{∂y} = \frac{1}{1 + [\frac{y^2}{x}]^2} * \frac{2y}{x}[/itex]

Once again, subbing in values;

[itex]\frac{∂f}{∂y} = \frac{1}{1 + [\frac{4}{√5}]^2} * \frac{-4}{√5} → \frac{-4}{√5( \frac{21}{5}) } [/itex]

→ [itex]\frac{∂f}{∂y} = \frac{-4}{√5(4\frac{1}{5}) } [/itex]

Pain in the butt, but that's fx, fy

----------------------------------------------------------------------------------------------------
Now, part B)

Find the rate of change of ∫ at the point (√5, -2) in the direction toward the point (0,0).

For directional vector I get (0 - √5, - (-2)) → (-√5, 2)

Then [itex] \frac{1} {√[(√5)^2 + (-2)^2]} → \frac {1}{√(5 + 4)} → \frac {1}{3}[/itex]

The unit vector is [itex] → \frac{1}{3}<-√5, 2>[/itex]

Now to get the Gradient, I, once again, take, [itex]\frac{∂f}{∂x} and \frac{∂f}{∂y}[/itex]

This will be :

[itex]\frac{∂f}{∂x} = \frac{- y^2}{x^2 + y^2}[/itex]

[itex]\frac{∂f}{∂y} = \frac{- 2y}{x(1 + \frac{y^4}{x^2})}[/itex]

[itex]∇∫ = <\frac{- y^2}{x^2 + y^2} , \frac{- 2y}{x(1 + \frac{y^4}{x^2})} > [/itex]

Thus

[itex]∇∫(√5, -2) = <\frac{- (-2)^2}{(√5)^2 + (-2)^2} , \frac{- 2(-2)}{√5(1 + \frac{(-2)^4}{(√5)^2})} > [/itex]

→ [itex]∇∫(√5, -2) = <\frac{-4}{5 + 4} , \frac{4}{√5(1 + \frac{16}{5})} > [/itex]

→ [itex]∇∫(√5, -2) = <\frac{-4}{9} , \frac{4}{√5(4\frac{1}{5})} > [/itex]

Since we have the unit vector and gradient, we use :

Du∫(√5, -2) = [itex]∇∫(√5, -2) \bullet u = <\frac{-4}{9} , \frac{4}{√5(4\frac{1}{5})} > \bullet \frac{1}{3}<-√5, 2> [/itex]

----------------------------------------------------------------------------------------------------

Now, since we have all that, onto part C.

Find the directional derivative in the direction of fastest increase at the point (√5, -2).

In reality this has already been solved by the above equation, as it will give us the rate of change as well. We just have to evaluate the equation and get.

Du∫(√5, -2) = [itex]∇∫(√5, -2) \bullet u = <\frac{-4}{9} , \frac{4}{√5(4\frac{1}{5})} > \bullet \frac{1}{3}<-√5, 2> [/itex]

[itex]\frac{1}{3} (\frac{4√5}{9}, \frac{8}{√5(4\frac{1}{5})})[/itex]


That should do it...what say you?

EDIT: Algebra fix...agaain
 
Last edited:
  • #7
Alex Bard said:
Thank you for the help LCKurtz, I'm going to go through the entire problem again, with feeling this time.[itex] Let f(x,y) = tan^-1(\frac{y^2}{x})[/itex]

A) Find fx(√5, -2) and fy(√5, -2).

[itex]\frac{∂f}{∂x} = \frac{1}{1 + [\frac{y^2}{x}]^2} * y^2 * \frac{-1}{x^2}[/itex]

Since I suck at simplification, I will plug my numbers in right away to avoid confusion:

Perhaps, if you plan to pass calculus, you should learn how to simplify algebraic expressions.

[itex]\frac{∂f}{∂x} = \frac{1}{1 + [\frac{(-2)^2}{√5}]^2} * (-2)^2 * \frac{-1}{(√5)^2}[/itex]

Solving:

[itex]\frac{∂f}{∂x} = \frac{1}{\frac{5}{5} + [\frac{16}{5}]} * 4 * \frac{-1}{5}[/itex]

→ [itex]\frac{∂f}{∂x} = \frac{-4}{21} [/itex]
Correct. If you can simplify it with numbers, you should be able to simplify it with variables.
Now for [itex]\frac{∂f}{∂y} [/itex]

[itex]\frac{∂f}{∂y} = \frac{1}{1 + [\frac{y^2}{x}]^2} * \frac{2y}{x}[/itex]

Once again, subbing in values;

[itex]\frac{∂f}{∂y} = \frac{1}{1 + [\frac{4}{√5}]^2} * \frac{-4}{√5} → \frac{-4}{√5( \frac{21}{5}) } [/itex]

→ [itex]\frac{∂f}{∂y} = \frac{-4}{√5(4\frac{1}{5}) } [/itex]

Pain in the butt, but that's fx, fy

Surely your teacher would dock points for an answer like that, right or wrong. And it is wrong.
[Edit]: Perhaps not wrong, just poorly presented. See my next post

----------------------------------------------------------------------------------------------------
Now, part B)

Find the rate of change of ∫ at the point (√5, -2) in the direction toward the point (0,0).

For directional vector I get (0 - √5, - (-2)) → (-√5, 2)

Then [itex] \frac{1} {√[(√5)^2 + (-2)^2]} → \frac {1}{√(5 + 4)} → \frac {1}{3}[/itex]

The unit vector is [itex] → \frac{1}{3}<-√5, 2>[/itex]

Now to get the Gradient, I, once again, take, [itex]\frac{∂f}{∂x} and \frac{∂f}{∂y}[/itex]

This will be :

[itex]\frac{∂f}{∂x} = \frac{- y^2}{x^2 + y^2}[/itex]

[itex]\frac{∂f}{∂y} = \frac{- 2y}{x(1 + \frac{y^4}{x^2})}[/itex]

[itex]∇∫ = <\frac{- y^2}{x^2 + y^2} , \frac{- 2y}{x(1 + \frac{y^4}{x^2})} > [/itex]

Thus

[itex]∇∫(√5, -2) = <\frac{- (-2)^2}{(√5)^2 + (-2)^2} , \frac{- 2(-2)}{√5(1 + \frac{(-2)^4}{(√5)^2})} > [/itex]

→ [itex]∇∫(√5, -2) = <\frac{-4}{5 + 4} , \frac{4}{√5(1 + \frac{16}{5})} > [/itex]

→ [itex]∇∫(√5, -2) = <\frac{-4}{9} , \frac{4}{√5(4\frac{1}{5})} > [/itex]

Since we have the unit vector and gradient, we use :

Du∫(√5, -2) = [itex]∇∫(√5, -2) \bullet u = <\frac{-4}{9} , \frac{4}{√5(4\frac{1}{5})} > \bullet \frac{1}{3}<-√5, 2> [/itex]

Doesn't it bother you that you don't have the same value for ##f_x## that you have earlier. And surely you wouldn't leave the answer like that. And what's with the integral signs?

----------------------------------------------------------------------------------------------------

Now, since we have all that, onto part C.

Find the directional derivative in the direction of fastest increase at the point (√5, -2).

In reality this has already been solved by the above equation, as it will give us the rate of change as well. We just have to evaluate the equation and get.

Du∫(√5, -2) = [itex]∇∫(√5, -2) \bullet u = <\frac{-4}{9} , \frac{4}{√5(4\frac{1}{5})} > \bullet \frac{1}{3}<-√5, 2> [/itex]

[itex]\frac{1}{3} (\frac{4√5}{9}, \frac{8}{√5(4\frac{1}{5})})[/itex]That should do it...what say you?

I say that last part is wrong. A directional derivative is a scalar and you have an answer that is a vector. Also there is something in your text about the direction of the max rate of change and its magnitude.
 
Last edited:
  • #8
##<\frac{-4}{9} , \frac{4}{√5(4\frac{1}{5})}> ##

I just realized that you apparently attempted to use a mixed fraction in that expression. I don't think they are ever used in printed material. In usual notation ##4\frac 1 5## is the same as ##4\cdot \frac 1 5 =\frac 4 5##. If you must use that notation, you would have to write it as ##4 + \frac 1 5##. And in any case you should simplify the expression.
 
  • #9
Alex Bard said:
Thank you for the help LCKurtz, I'm going to go through the entire problem again, with feeling this time.


[itex] Let f(x,y) = tan^-1(\frac{y^2}{x})[/itex]

A) Find fx(√5, -2) and fy(√5, -2).

[itex]\frac{∂f}{∂x} = \frac{1}{1 + [\frac{y^2}{x}]^2} * y^2 * \frac{-1}{x^2}[/itex]

Since I suck at simplification, I will plug my numbers in right away to avoid confusion:

[itex]\frac{∂f}{∂x} = \frac{1}{1 + [\frac{(-2)^2}{√5}]^2} * (-2)^2 * \frac{-1}{(√5)^2}[/itex]

Solving:

[itex]\frac{∂f}{∂x} = \frac{1}{\frac{5}{5} + [\frac{16}{5}]} * 4 * \frac{-1}{5}[/itex]

→ [itex]\frac{∂f}{∂x} = \frac{-4}{21} [/itex]

Now for [itex]\frac{∂f}{∂y} [/itex]

[itex]\frac{∂f}{∂y} = \frac{1}{1 + [\frac{y^2}{x}]^2} * \frac{2y}{x}[/itex]

Once again, subbing in values;

[itex]\frac{∂f}{∂y} = \frac{1}{1 + [\frac{4}{√5}]^2} * \frac{-4}{√5} → \frac{-4}{√5( \frac{21}{5}) } [/itex]

→ [itex]\frac{∂f}{∂y} = \frac{-4}{√5(4\frac{1}{5}) } [/itex]

Pain in the butt, but that's fx, fy

----------------------------------------------------------------------------------------------------
Now, part B)

Find the rate of change of ∫ at the point (√5, -2) in the direction toward the point (0,0).

For directional vector I get (0 - √5, - (-2)) → (-√5, 2)

Then [itex] \frac{1} {√[(√5)^2 + (-2)^2]} → \frac {1}{√(5 + 4)} → \frac {1}{3}[/itex]

The unit vector is [itex] → \frac{1}{3}<-√5, 2>[/itex]

Now to get the Gradient, I, once again, take, [itex]\frac{∂f}{∂x} and \frac{∂f}{∂y}[/itex]

This will be :

[itex]\frac{∂f}{∂x} = \frac{- y^2}{x^2 + y^2}[/itex]

[itex]\frac{∂f}{∂y} = \frac{- 2y}{x(1 + \frac{y^4}{x^2})}[/itex]

[itex]∇∫ = <\frac{- y^2}{x^2 + y^2} , \frac{- 2y}{x(1 + \frac{y^4}{x^2})} > [/itex]

Thus

[itex]∇∫(√5, -2) = <\frac{- (-2)^2}{(√5)^2 + (-2)^2} , \frac{- 2(-2)}{√5(1 + \frac{(-2)^4}{(√5)^2})} > [/itex]

→ [itex]∇∫(√5, -2) = <\frac{-4}{5 + 4} , \frac{4}{√5(1 + \frac{16}{5})} > [/itex]

→ [itex]∇∫(√5, -2) = <\frac{-4}{9} , \frac{4}{√5(4\frac{1}{5})} > [/itex]

Since we have the unit vector and gradient, we use :

Du∫(√5, -2) = [itex]∇∫(√5, -2) \bullet u = <\frac{-4}{9} , \frac{4}{√5(4\frac{1}{5})} > \bullet \frac{1}{3}<-√5, 2> [/itex]

----------------------------------------------------------------------------------------------------

Now, since we have all that, onto part C.

Find the directional derivative in the direction of fastest increase at the point (√5, -2).

In reality this has already been solved by the above equation, as it will give us the rate of change as well. We just have to evaluate the equation and get.

Du∫(√5, -2) = [itex]∇∫(√5, -2) \bullet u = <\frac{-4}{9} , \frac{4}{√5(4\frac{1}{5})} > \bullet \frac{1}{3}<-√5, 2> [/itex]

[itex]\frac{1}{3} (\frac{4√5}{9}, \frac{8}{√5(4\frac{1}{5})})[/itex]


That should do it...what say you?

EDIT: Algebra fix...agaain

You may "suck" at simplification, but that just means that your elementary algebra skills are deficient. That is not a good sign if you plan to continue in mathematics, physics, operations research, engineering, etc. NOW is the time to improve your algebraic skills; perhaps working through one of the "College Outline" books, or using some other tutorial packages on- or off-line is what you need to do. Life will get increasingly difficult for you if you don't address this issue.
 
  • #10
Perhaps, if you plan to pass calculus, you should learn how to simplify algebraic expressions.

Thank you for your input, I will work in variables more to get that buttoned up.

Surely your teacher would dock points for an answer like that, right or wrong. And it is wrong.

Would you please give me a hint? I'm not exactly sure where my mistake is.

Looking at it again I can see it this way.

tan^-1(y^2/x)

∂z/∂y = [itex]\frac{1}{1 + (\frac{y^2}{x})^2} * \frac{2y}{x} [/itex]

→ [itex]\frac{2y}{1 + \frac{y^4}{x^2}} * \frac{1}{x}[/itex]

[itex]\frac{2y}{x(1 + y^2x^{-2})}[/itex]

Here I can distribute x but that's about it as far as I can tell and when I solve, I will get [itex]\frac{-4}{√5(1 + 4(5^{-1}))} → \frac{-4√5}{9}[/itex].
Doesn't it bother you that you don't have the same value for fx that you have earlier.

That was an absent minded mistake, it should have been a y^4 in the denominator, giving me the same answer.

[itex]∂z/∂x = \frac {-4}{21} [/itex]
[itex]∂z/∂y = \frac {-4√5}{9} [/itex]

My unit vector is [itex]\frac{1}{3}<-√5, 2>[/itex]

Dotting the vectors then adding I get:

Duf = [itex](\frac {-4}{21})(\frac{-√5}{3}) + (\frac {-4√5}{9})(\frac{2}{3}) → \frac{4√5}{63} - \frac{8√5}{27} → -\frac{44√5}{189}[/itex]

Rate of change of f in the direction from (√5, -2) to (0, 0) is [itex]-\frac{44√5}{189}[/itex]

**We are not allowed to use calculators in my school so this answer should be fine**

I say that last part is wrong. A directional derivative is a scalar and you have an answer that is a vector. Also there is something in your text about the direction of the max rate of change and its magnitude.

I'm a little lost really. I know that the direction of the fastest increase is the gradient vector. I also know that the directional derivative formula is Duf = ∇f°u but I really don't know how to apply it.
 
  • #11
You may "suck" at simplification, but that just means that your elementary algebra skills are deficient. That is not a good sign if you plan to continue in mathematics, physics, operations research, engineering, etc. NOW is the time to improve your algebraic skills; perhaps working through one of the "College Outline" books, or using some other tutorial packages on- or off-line is what you need to do. Life will get increasingly difficult for you if you don't address this issue.

Thank you for the input - as I understand how important it is, I will surely be working on it.
 
  • #12
Alex Bard said:
Thank you for your input, I will work in variables more to get that buttoned up.

Would you please give me a hint? I'm not exactly sure where my mistake is.

Looking at it again I can see it this way.

tan^-1(y^2/x)

∂z/∂y = [itex]\frac{1}{1 + (\frac{y^2}{x})^2} * \frac{2y}{x} [/itex]

→ [itex]\frac{2y}{1 + \frac{y^4}{x^2}} * \frac{1}{x}[/itex]

[itex]\frac{2y}{x(1 + y^{\color{red}{2}}x^{-2})}[/itex]

Here I can distribute x but that's about it as far as I can tell

As long as you have negative exponents in there like that, it isn't simplified. Fix the red typo and multiply both the numerator and denominator by ##x##, then distribute the ##x^2## in the denominator. You should get$$
\frac {2xy}{x^2+y^4}$$

and when I solve, I will get [itex]\frac{-4}{√5(1 + 4(5^{-1}))} → \frac{-4√5}{9}[/itex].

Try again with the simplified expression. Simplifying makes things easier.

That was an absent minded mistake, it should have been a y^4 in the denominator, giving me the same answer.

[itex]∂z/∂x = \frac {-4}{21} [/itex]
[itex]∂z/∂y = \frac {-4√5}{9} [/itex]

My unit vector is [itex]\frac{1}{3}<-√5, 2>[/itex]

Dotting the vectors then adding I get:

Duf = [itex](\frac {-4}{21})(\frac{-√5}{3}) + (\frac {-4√5}{9})(\frac{2}{3}) → \frac{4√5}{63} - \frac{8√5}{27} → -\frac{44√5}{189}[/itex]

Rate of change of f in the direction from (√5, -2) to (0, 0) is [itex]-\frac{44√5}{189}[/itex]

**We are not allowed to use calculators in my school so this answer should be fine**

Again, do it with the correct numbers.

I'm a little lost really. I know that the direction of the fastest increase is the gradient vector.

Yes. And what about the gradient gives the max rate of change?
 
  • #13
Where should I send the cake?

LCKurtz, I believe I owe you a cup of coffee or something. Thank you for your input and gentle guidance. [itex] Let f(x, y) = tan^{-1} \frac{y^2}{x} [/itex]

A.

[itex]fx(√5, -2) = \frac{1}{1 + \frac{y^4}{x^2}} * \frac{y^2}{-x^2}

→ \frac{-y^2}{1 + y^4x^{-2}} * \frac{1}{x^2} → \frac{-y^2}{x^2 + y^4} → \frac{-4}{21}
[/itex]

[itex]
fy(√5, -2) = \frac{1}{1 + \frac{y^4}{x^2}} * \frac{2y}{x}

→ \frac{2y}{1 + y^4x^{-2}} * \frac{1}{x} *\frac{x}{x} → \frac{2yx}{x^2 + y^4} → \frac{-4√5}{21}

[/itex]

B.

P = (√5, -2) O = (0, 0) → PO = <-√5, 2>

u = √[(-√5)^2 + 2^2] → √(5 + 4) = 3 → [itex]\frac{1}{3}<-√5, 2> → <\frac{-√5}{3}, \frac{2}{3}>[/itex]

∇f(x, y) = <fx, fy> = [itex]<\frac{-y^2}{x^2 + y^4}, \frac{2yx}{x^2 + y^4}>[/itex]

∇f(√5, -2) = [itex]<\frac{-4}{21}, \frac{-4√5}{21}>[/itex]

The rate of Change of f in the Direction P to Q is:
Duf(√5, -2) = [itex]<\frac{-4}{21}, \frac{-4√5}{21}> \bullet <\frac{-√5}{3}, \frac{2}{3}> = (\frac{4√5}{63} - \frac{8√5}{63}) = \frac{-4√5}{63}
[/itex]

C.

The Directional Derivative is ∇f(x, y) [itex]\bullet[/itex] u and since θ = 0 because u is pointing in the same direction as the gradient vector, it is the maximum rate of increase.

∇f(x, y) [itex]\bullet[/itex] u =

[itex]<\frac{-4}{21}, \frac{-4√5}{21}> \bullet <\frac{-√5}{3}, \frac{2}{3}>cosθ = (\frac{4√5}{63} - \frac{8√5}{63}) (1) = \frac{-4√5}{63}
[/itex]
 
Last edited:
  • #14
Alex Bard said:
LCKurtz, I believe I owe you a cup of coffee or something. Thank you for your input and gentle guidance.


[itex] Let f(x, y) = tan^{-1} \frac{y^2}{x} [/itex]

A.

[itex]fx(√5, -2) = \frac{1}{1 + \frac{y^4}{x^2}} * \frac{y^2}{-x^2}

→ \frac{-y^2}{1 + y^4x^{-2}} * \frac{1}{x^2} → \frac{-y^2}{x^2 + y^4} → \frac{-4}{25}
[/itex]

[itex]
fy(√5, -2) = \frac{1}{1 + \frac{y^4}{x^2}} * \frac{2y}{x}

→ \frac{2y}{1 + y^4x^{-2}} * \frac{1}{x} *\frac{x}{x} → \frac{2yx}{x^2 + y^4} → \frac{-4√5}{25}

[/itex]

Better check those denominators. I don't get ##25##.

B.

P = (√5, -2) O = (0, 0) → PO = <-√5, 2>

u = √[(-√5)^2 + 2^2] → √(5 + 4) = 3 → [itex]\frac{1}{3}<-√5, 2> → <\frac{-√5}{3}, \frac{2}{3}>[/itex]

∇f(x, y) = <fx, fy> = [itex]<\frac{-y^2}{x^2 + y^4}, \frac{2yx}{x^2 + y^4}>[/itex]

∇f(√5, -2) = [itex]<\frac{-4}{25}, \frac{-4√5}{25}>[/itex]

The rate of Change of f in the Direction P to Q is:
Duf(√5, -2) = [itex]<\frac{-4}{25}, \frac{-4√5}{25}> \bullet <\frac{-√5}{3}, \frac{2}{3}> = (\frac{4√5}{75} - \frac{8√5}{75}) = \frac{-4√5}{75}
[/itex]

C.

The Directional Derivative is ∇f(x, y) [itex]\bullet[/itex] u and since θ = 0 because u is pointing in the same direction as the gradient vector, it is the maximum rate of increase.

∇f(x, y) [itex]\bullet[/itex] u =

[itex]<\frac{-4}{25}, \frac{-4√5}{25}> \bullet <\frac{-√5}{3}, \frac{2}{3}>cosθ = (\frac{4√5}{75} - \frac{8√5}{75}) (1) = \frac{-4√5}{75}
[/itex]

The vector u doesn't have anything to do with the last part. What does your book say about the gradient vs. max rate of change. Look it up.
 
  • #15
I fixed the 25 to 21.

This is the theorem from the book. I actually didn't feel too comfortable with the answer myself and have been searching the internet for a better explanation. I understand that the gradient is the direction of the maximum increase, and is orthogonal to the level curve/surface. I also understand that it measures just one maximum and not all.

Now as far as a directional derivative, I've been reading here and trying to understand as I go.

Its frustrating to think you almost have it but there is a little bit of information I'm just not getting. Thankfully with persistence I know I will, the AHA is just around the corner.
 

Attachments

  • Screen Shot 2013-11-03 at 8.25.18 PM.png
    Screen Shot 2013-11-03 at 8.25.18 PM.png
    7.1 KB · Views: 430
  • #17
Okay sir, I'm stumped.

Find the Directional derivative in the direction of fastest increase at the point (√5, -2)

The Direction of the FASTEST increase is Gradient, which I've found is ∇f(√5, -2) = [itex]<\frac{-4}{21}, \frac{-4√5}{21}>[/itex]

The magnitude of the gradient is the maximal directional derivative. The directional derivative is maximal in the direction of [itex]<\frac{-4}{21}, \frac{-4√5}{21}>[/itex]

So that means I would have to put [itex]\frac{-4}{21}, \frac{-4√5}{21} over √{(\frac{-4}{21})^2 + (\frac{-4√5}{21})^2[/itex], correct?

and wouldn't that just return my unit vector from before? or am I taking the wrong point here?
 
  • #18
Alex Bard said:
The magnitude of the gradient is the maximal directional derivative. The directional derivative is maximal in the direction of [itex]<\frac{-4}{21}, \frac{-4√5}{21}>[/itex]
Yes, but that's not just the direction. It is ∇f, and according to the theorem you just want the magnitude of that.
 
  • #19
So then it's simply square both and add under a square root?
 
  • #20
Alex Bard said:
So then it's simply square both and add under a square root?

That's my understanding
 
  • Like
Likes 1 person
  • #21
Alex Bard said:
So then it's simply square both and add under a square root?

Yes. The gradient points the direction of max increase. The magnitude (length) of the gradient vector is the value of the max rate of change.
 
  • Like
Likes 1 person
  • #22
Thank you! Finally I can put this problem to rest, I believe. Guess I'll have to look at it with fresh eyes in the morning.
 

1. What is a gradient?

A gradient is a vector that represents the direction and rate of change of a function at a specific point. It is calculated by taking the partial derivatives of the function with respect to each independent variable.

2. How is a gradient used in directional derivatives?

A gradient is used to find the direction of steepest increase of a function at a specific point. The directional derivative is then calculated by taking the dot product of the gradient and a unit vector in the desired direction.

3. Can the gradient be negative?

Yes, the gradient can be negative. A negative gradient indicates a decrease in the function in the direction of the gradient, while a positive gradient indicates an increase in the function.

4. What is the relationship between the gradient and level curves?

The gradient is perpendicular to the level curves of a function. This means that the gradient points in the direction of the steepest increase of the function, while the level curves represent points with the same function value.

5. How do you find the maximum and minimum values using the gradient?

The maximum and minimum values of a function can be found by setting the gradient equal to zero and solving for the independent variables. This will give the critical points, which can then be evaluated to determine if they are maximum or minimum points.

Similar threads

  • Calculus and Beyond Homework Help
Replies
8
Views
466
  • Calculus and Beyond Homework Help
Replies
4
Views
559
  • Calculus and Beyond Homework Help
Replies
2
Views
503
Replies
9
Views
708
  • Calculus and Beyond Homework Help
Replies
6
Views
847
  • Calculus and Beyond Homework Help
Replies
21
Views
2K
  • Calculus and Beyond Homework Help
Replies
5
Views
758
  • Calculus and Beyond Homework Help
Replies
10
Views
429
  • Calculus and Beyond Homework Help
Replies
2
Views
536
  • Calculus and Beyond Homework Help
Replies
6
Views
545
Back
Top